- PowerScore Staff
- Posts: 5972
- Joined: Mar 25, 2011
- Mon Jan 20, 2014 12:00 am
#43584
Complete Question Explanation
(The complete setup for this game can be found here: lsat/viewtopic.php?t=8007)
The correct answer choice is (B)
The correct answer choice is (B) W and Y. If a W and a Y were both selected, then G, H, and K would all be precluded from selection, since G cannot go with a Y, and H and K both need an X. This would leave only J and L as possible fish selections, which is unacceptable, because exactly three fish must be selected.
One effective method of attack on this question is to use the information from other questions and answer choices. For example, the information from question #12 can be used to instantly eliminate answer choice (A), and the information from question #14 can be used to instantly eliminate answer choice (E).
(The complete setup for this game can be found here: lsat/viewtopic.php?t=8007)
The correct answer choice is (B)
The correct answer choice is (B) W and Y. If a W and a Y were both selected, then G, H, and K would all be precluded from selection, since G cannot go with a Y, and H and K both need an X. This would leave only J and L as possible fish selections, which is unacceptable, because exactly three fish must be selected.
One effective method of attack on this question is to use the information from other questions and answer choices. For example, the information from question #12 can be used to instantly eliminate answer choice (A), and the information from question #14 can be used to instantly eliminate answer choice (E).
Dave Killoran
PowerScore Test Preparation
Follow me on X/Twitter at http://twitter.com/DaveKilloran
My LSAT Articles: http://blog.powerscore.com/lsat/author/dave-killoran
PowerScore Podcast: http://www.powerscore.com/lsat/podcast/
PowerScore Test Preparation
Follow me on X/Twitter at http://twitter.com/DaveKilloran
My LSAT Articles: http://blog.powerscore.com/lsat/author/dave-killoran
PowerScore Podcast: http://www.powerscore.com/lsat/podcast/